2016
DOI: 10.1093/qmath/haw005
|View full text |Cite
|
Sign up to set email alerts
|

Lower Bounds for the Variance of Sequences in Arithmetic Progressions: Primes and Divisor Functions

Abstract: Abstract. We develop a general method for lower bounding the variance of sequences in arithmetic progressions mod q, summed over all q ≤ Q, building on previous work of Liu, Perelli, Hooley, and others. The proofs lower bound the variance by the minor arc contribution in the circle method, which we lower bound by comparing with suitable auxiliary exponential sums that are easier to understand.As an application, we prove a lower bound of (1 − ǫ)QN log(Q 2 /N ) for the variance of the von Mangoldt function (Λ(n)… Show more

Help me understand this report

Search citation statements

Order By: Relevance

Paper Sections

Select...
3
2

Citation Types

0
29
0

Year Published

2021
2021
2023
2023

Publication Types

Select...
5

Relationship

0
5

Authors

Journals

citations
Cited by 12 publications
(29 citation statements)
references
References 25 publications
0
29
0
Order By: Relevance
“…This is essentially due to the fact that the peaks of the exponential sum with coefficients f (n) over small neighbourhoods of such fractions might be amplified by the effect of a higher power inside the integral. The lower bound (1.2) has been proven to hold when f (n) = (n) and f (n) = d k (n), for k ≥ 2 a positive integer and Q in the range N 1/2+δ ≤ Q ≤ N , for any small δ > 0, by Theorems 1 and 2 in [12]. Here (n) indicates as usual the Von Mangoldt function.…”
Section: Statement Of the Main Resultsmentioning
confidence: 93%
See 4 more Smart Citations
“…This is essentially due to the fact that the peaks of the exponential sum with coefficients f (n) over small neighbourhoods of such fractions might be amplified by the effect of a higher power inside the integral. The lower bound (1.2) has been proven to hold when f (n) = (n) and f (n) = d k (n), for k ≥ 2 a positive integer and Q in the range N 1/2+δ ≤ Q ≤ N , for any small δ > 0, by Theorems 1 and 2 in [12]. Here (n) indicates as usual the Von Mangoldt function.…”
Section: Statement Of the Main Resultsmentioning
confidence: 93%
“…The present paper studies the question of finding a lower bound for the quantity V (Q, f ) when considering suitable generalizations f of the divisor functions introduced above. The main reference on this problem is the Harper and Soundararajan's paper [12], in which the authors set up the bases for the study of lower bounds of variances of complex sequences in arithmetic progressions. More precisely, they showed that for a wide class of functions with a controlled growth we can lower bound the variance (1.1) with the L 2 -norm of the exponential sum with coefficients f (n) over a large portion of the circle, namely, the so-called union of minor arcs.…”
Section: Statement Of the Main Resultsmentioning
confidence: 99%
See 3 more Smart Citations